¿Son los campos magnéticos simplemente campos eléctricos relativistas modificados?

Feynman's Lectures, volumen 2, dice que la fuerza electromagnética es invariable en cualquier marco de referencia, y la fuerza magnética en un marco se convierte en el campo eléctrico en otro.

Y Wikipedia dice :

Es decir, el campo magnético es simplemente el campo eléctrico, como se ve en un sistema de coordenadas en movimiento.

¿Podemos entonces decir que el campo magnético es solo una versión "relativista" modificada del campo eléctrico?

Lea esto y los enlaces dentro de physics.stackexchange.com/q/24010/104696
No. El campo electromagnético es inherentemente relativista. No existe una "versión no relativista" del campo electromagnético. El campo se crea moviendo cargas. Las cargas que se mueven en el espacio crean un campo magnético. Las cargas que se mueven en el tiempo crean un campo eléctrico.
Estimado Safesphere, ¿puede decirme más sobre "Las cargas que se mueven en el tiempo crean un campo eléctrico".? Un objeto que no se mueve a la velocidad c, por lo que moverse más lento siempre se mueve en la dimensión del tiempo. Entonces, una carga (que se mueve más lentamente que c en el espacio) siempre se mueve en el tiempo. Cualquier cosa con masa en reposo (y la carga tiene que tener masa en reposo), siempre es así, ¿entonces una carga siempre crea un campo eléctrico?
@safesphere Define "Cargas que se mueven en el tiempo". Además, las cargas en movimiento ("en el espacio" obviamente, donde más) causan un campo eléctrico y magnético.
@OP "Feynman ... dijo que la fuerza electromagnética es invariable en cualquier marco de referencia". Dijo que la fuerza electromagnética es covariante de Lorentz, que se transforma como un vector de cuatro.
@ÁrpádSzendrei Sí, tu lógica es completamente correcta. Una carga puede moverse solo en el tiempo (en su propio marco de reposo) o puede moverse tanto en el tiempo como en el espacio (en cualquier otro marco). Nunca sólo en el espacio. Por lo tanto, una carga puede o no crear un campo magnético, pero siempre crea un campo eléctrico. Un movimiento de carga es actual. Cuando combina movimientos en el espacio y el tiempo, obtiene 4 corrientes (densidad): en.m.wikipedia.org/wiki/Four-current - La corriente 4 tiene 3 componentes espaciales que reflejan el movimiento de la carga en el espacio y 1 componente temporal que refleja el movimiento de la carga en el tiempo.
@my2cts Consulte mi último comentario.
@safesphere "Por favor, vea mi último comentario" Lo hice y noté declaraciones indefinidas y también poco claras, posibles errores. Estas son las que me gustaría que aclararas.
¡Gracias por los útiles comentarios a todos! Pero, ¿sigue siendo correcto decir que un campo magnético se forma al considerar algún campo eléctrico en un cierto marco de referencia?
@ my2cts Quiero decir, vea mi respuesta a Árpád Szendrei. Hay un enlace Wiki allí al artículo actual 4 que describe los conceptos que mencioné.
Dudo que Feynman dijera que la fuerza de Lorentz es invariante, porque, como otros vectores, la fuerza no puede ser una invariante relativista.

Respuestas (1)

ingrese la descripción de la imagen aquí


En la Figura-02 anterior, un sistema inercial S se traduce con respecto al sistema inercial S con velocidad constante

(01) υ = ( υ 1 , υ 2 , υ 3 ) = ( υ norte 1 , υ norte 2 , υ norte 3 ) = υ norte , υ ( C , C )
La transformación de Lorentz es
(02a) X = X + ( γ 1 ) ( norte X ) norte γ υ t (02b) t = γ ( t υ X C 2 )
en forma diferencial
(03a) d X = d X + ( γ 1 ) ( norte d X ) norte γ υ d t (03b) d t = γ ( d t υ d X C 2 )
y en forma matricial
(04) X = [ X γ υ C C t γ υ C ] = [ I + ( γ 1 ) norte norte γ υ C γ υ C γ υ C γ ] [ X γ υ C C t γ υ C ] = L X
dónde L el verdadero simétrico 4 × 4 matriz
(05) L [ I + ( γ 1 ) norte norte γ υ C γ υ C γ υ C γ ]
y
(06) norte norte = [ norte 1 norte 2 norte 3 ] [ norte 1 norte 2 norte 3 ] = [ norte 1 2 norte 1 norte 2 norte 1 norte 3 norte 2 norte 1 norte 2 2 norte 2 norte 3 norte 3 norte 1 norte 3 norte 2 norte 3 2 ]

una matriz que representa la proyección vectorial en la dirección norte .

El campo electromagnético es un ente y esto queda más claro si echamos un vistazo a su transformación. Entonces, para la transformación de Lorentz (02), los vectores mi y B se transforman de la siguiente manera

(07a) mi = γ mi ( γ 1 ) ( mi norte ) norte + γ ( υ × B ) (07b) B = γ B ( γ 1 ) ( B norte ) norte γ C 2 ( υ × mi )
Ahora, deje que la fuerza de Lorentz 3-vector en una partícula puntual con carga q moviéndose con velocidad tu con respecto a S
(08) F = q ( mi + tu × B )
Esta fuerza de Lorentz de 3 vectores con respecto a S es
(09) F = q ( mi + tu × B )
Tenga en cuenta que el valor de la carga q de una partícula puntual es, por hipótesis, la misma en todos los sistemas inerciales (una invariante escalar), mientras que la velocidad de 3 vectores tu se transforma de la siguiente manera
(10) tu = tu + ( γ 1 ) ( norte tu ) norte γ υ γ ( 1 υ tu C 2 )
ecuación probada dividiendo las ecuaciones (03a), (03b) una al lado de la otra y estableciendo tu d X / d t , tu d X / d t .

Ahora, si en (09) reemplazamos mi , B , tu por sus expresiones (07a),(07b) y (10) respectivamente, entonces terminamos con la siguiente relación entre los 3-vectores de fuerza

(11) F = F + ( γ 1 ) ( norte F ) norte γ υ ( F tu C 2 ) γ ( 1 υ tu C 2 )
donde las cantidades del campo electromagnético D I S A PAG PAG mi A R mi D ! ! !

Es por eso que en los primeros años de la Relatividad Especial se creía que la transformación (11) era válida para cualquier fuerza al menos del mismo tipo que la fuerza de Lorentz (más exactamente para cualquier fuerza que no cambie la masa en reposo de la partícula).
Siguiendo el mismo camino por el que construimos a partir de (10) el cuadrivector de velocidad tu

(12) tu = ( γ tu tu , γ tu C )
construimos también a partir de (11) el vector de fuerza 4 F
(13) F = ( γ tu F , γ tu F tu C )
Lorentz transformado
(14) F = L F
o
(15) F = [ γ tu F γ υ C γ tu F tu C ] = [ I + ( γ 1 ) norte norte γ υ C γ υ C γ υ C γ ] [ γ tu F γ υ C γ tu F tu C ] = L F

Tal vez valga la pena mencionar que mi 2 B 2 es un invariante. Si es positivo, puedes encontrar la transformación que hace B de E. Si es negativo, viceversa.
@Bert Barrois Me disculpo, pero no entiendo a qué te refieres con tu comentario. Por favor aclare: de uno o ambos invariantes de Lorentz ( | | mi | | 2 C 2 | | B | | 2 ) y ( mi B ) podríamos producir una o ambas ecuaciones de transformación (07) ???
No hay absolutamente nada de malo en tu respuesta. Todo lo que digo es que las condiciones necesarias y suficientes para la existencia de una transformación de una combinación arbitraria de campos E y B hacia/desde un campo E puro son: mi B = 0 Y mi 2 B 2 > 0 . (Viceversa si B es más fuerte que E). El OP había visto una declaración que parecía decir que el campo B no es más que un campo E transformado, pero eso no siempre es posible.
me encanta el diagrama; estas usando Inkscape?
@Larry Harson: Gracias. Yo uso GeoGebra. Es gratis.